六年级英语上册期中试题(最新版)

合集下载

小学六年级英语上册期中考试及答案【完整版】

小学六年级英语上册期中考试及答案【完整版】

小学六年级英语上册期中考试及答案【完整版】题序一二三四五六七八九十总分得分(满分:100分时间:60分钟)(听力部分40分)一、听录音,选出你所听到的单词或短语,听两遍。

(10分)1、( ) A.happily B.happy C.happiness2. ( ) A.windy B.sunny C.rainy3. ( ) A.looking B.interesting C.interested4. ( ) A.flies away B.flying away C.fly away5. ( ) A.the first time B.the third time C.the second time二、听录音,给下列图片排序。

每小题读两遍。

(10分)( )( )( )( ) ( )三、听录音,选出你所听到的句子,读两遍。

(10分)( ) 1. A.Have you got any pencils? B.Have you got any pens?( ) 2. A.I have got toy dolls. B.I have got toy cars.( ) 3. A.Collecting stamps is my hobby. B.Collecting toys is my hobby. ( ) 4. A.What are these? B.What are those?( ) 5. A.Flying kites is my hobby. B.Flying kites is his hobby.四、听录音,选出正确的答句,每小题读两遍。

(10分)( ) 1. A. I wear size 35. B.My size is small. ( ) 2. A.I went to Turpan. B.I’m OK now.( ) 3. A.I hurt the leg. B.I am taller than you. ( ) 4. A.I looked it up on the Internet. B.No, I don’t.( ) 5. A.Yes, you did. B.No, I didn’t.(笔试部分60分)五、单项选择,将正确答案序号填在括号里。

小学六年级上册第5次英语第四单元期中试卷(答案和解释)

小学六年级上册第5次英语第四单元期中试卷(答案和解释)

小学六年级上册英语第四单元期中试卷(答案和解释)英语试题一、综合题(本题有50小题,每小题2分,共100分.每小题不选、错误,均不给分)1.Tom and his family are visiting the __________. They see many different kinds of __________, including elephants, tigers, and lions. Tom’s little sister loves the__________ because it has big ears and a long trunk. After looking at the animals, they sit down to have some __________ and enjoy the sunny weather.2.What do you use to write on paper?A. PencilB. BrushC. HammerD. Spoon3.What do you use to write on paper?A. EraserB. ScissorsC. PenD. Fork4.Which of these is used to cut food?A. KnifeB. ForkC. SpoonD. Plate5.My family loves to travel. Last summer, we went to a beach for a vacation. We stayed in a nice hotel near the __. Every morning, we went swimming in the __. In the afternoon, we played volleyball and built sandcastles. In the evening, we walked along the __ and enjoyed the sunset. We also tried some delicious local __. I can’t wait for our next trip!6.Which one is a number?A. SevenB. SpoonC. PlateD. Book7.Which one is a day of the week?A. MondayB. SpoonC. PlateD. Chair8.Jack and his family are at the beach. The weather is __________ and the sun is shining brightly. Jack wears his __________ and runs to the water. He likes to play in the__________ water. After swimming, Jack builds a __________ in the sand and collects some __________. He has a lot of fun at the beach with his family.9.Every afternoon, I _______ (do) my homework after school. Then, I _______ (play) with my friends in the park. We _______ (ride) bikes, play soccer, and _______ (talk) about our favorite games.10.Which animal lives in the ocean?A. DogB. FishC. ElephantD. Tiger11.My aunt _______ (teach) us how to cook. She _______ (show) us how to make delicious cookies. I _______ (help) her mix the ingredients, and then we _______ (bake) them.12.Which of these is used for cleaning the floor?A. BroomB. PlateC. ChairD. Towel13.Which one is a tool?A. BookB. HammerC. ChairD. Apple14.I ______ (love) reading books. Every weekend, I ______ (go) to the library with my sister. Last Saturday, we ______ (find) a new book about dinosaurs. It ______ (be) very interesting. We ______ (read) it together, and we ______ (learn) many new things.15.Which one is a number?A. TenB. SpoonC. PlateD. Fork16.How do you say "谢谢" in English?A. Excuse meB. PleaseC. Thank youD. Sorryst summer, I ______ (go) to the beach with my family. We ______ (swim) in the sea and ______ (build) sandcastles. My little brother ______ (enjoy) playing with the sand, but I ______ (like) swimming more. After the beach, we ______ (have) ice cream, and it ______ (be) delicious.18.Which of these is a pet animal?A. DogB. WhaleC. ElephantD. Lion19.My mom ______ (make) dinner every evening. She ______ (usually/cook) rice, vegetables, and chicken. Yesterday, she ______ (make) spaghetti, which I ______ (love). After dinner, we ______ (watch) a movie together. I ______ (always/feel) happy when we ______ (spend) time as a family.20.What is the opposite of "fast"?A. SlowB. QuickC. RapidD. Speedy21.What do you use to write?A. PencilB. SpoonC. ForkD. Plate22.Jack loves watching movies. His favorite type of movie is __________. He especially enjoys watching __________ movies because he loves action scenes. This weekend, he is going to the theater with his friends to see the new __________ movie. Jack can’t wait to watch it!st weekend, I ______ (go) to the park with my friends. We ______ (play) soccer and ______ (have) a picnic under a big tree. I ______ (take) some photos because the weather ______ (be) so nice. After we ______ (finish) our picnic, we ______ (walk) around the park and ______ (see) some animals in the zoo.24.Mary _______ (study) very hard for her English test. She _______ (review) her notes every night before the exam. Yesterday, she _______ (finish) her homework early so she _______ (have) more time to relax.25.What do we use to brush our teeth?A. ToothbrushB. SoapC. TowelD. Comb26.What is the opposite of "up"?A. DownB. LeftC. RightD. Middle27.Which animal is known for its ability to swim and live in the water?A. DogB. FishC. TigerD. Bird28.What do we use to eat ice cream?A. KnifeB. SpoonC. ForkD. Plate29.What color is the grass?A. RedB. GreenC. YellowD. Blue30.My school is very big. It ______ (have) a playground, a library, and a computer room. Every morning, we ______ (start) class at 8:30. Our teacher ______ (be) very kind, and we ______ (enjoy) learning with her.31.Which animal is called the "king of the jungle"?A. TigerB. LionC. ElephantD. Bear32.My school is very __________. We have a large __________ with many different __________. I like to go to the __________ after school to read books. There is also a__________ where we play basketball. My favorite subject is __________ because I love learning about the world. I study __________ every day, and I try my best to__________.33.Every morning, I wake up at __________ (1) o’clock. I br ush my __________ (2) and wash my __________ (3). Then, I eat __________ (4) for breakfast, which is my favorite food. After that, I put on my __________ (5) and take my __________ (6) to school. It takes me __________ (7) minutes to get to school. I like my school because it is very __________ (8).34.Which of these is a shape with three sides?A. CircleB. TriangleC. SquareD. Rectangle35.What is the color of an apple?A. RedB. GreenC. BlueD. Purple36.Which of these is a fruit?A. PotatoB. TomatoC. OnionD. Cucumber37.Your friend is upset because they lost their favorite toy. What is the best way to comfort them?A. Laugh at them and tell them it’s not a big dealB. Ignore your friend and pretend you don’t careC. Offer to help them search for the toy and show empathyD. Tell them it’s just a toy and they will forget about it38.I ______ (be) very tired yesterday because I ______ (study) for a long time. I______ (go) to bed early. This morning, I ______ (feel) much better. I ______ (eat) a healthy breakfast and ______ (go) to school.39.Which of these is a body part?A. ChairB. LegC. PlateD. Spoon40.Which of the following is a vegetable?A. BananaB. CarrotC. FishD. Egg41.I have a pet __ named Max. He is very friendly and loves to play. Every day, I feed him some __ and give him fresh __. Max loves to run around the garden, but he also likes to sleep in his cozy __. He’s always excited when I come home from school, and we play together in the __ after school.42.Which of these is used to measure time?A. RulerB. WatchC. SpoonD. Chair43.Which one is a kind of animal?A. DogB. ChairC. SpoonD. Plate44.Which of these is a mode of transport?A. BoatB. DogC. TreeD. Plate45.I __________ (1) a new bike last week. It __________ (2) blue and white, and it __________ (3) very fast. My dad __________ (4) it for me. We __________ (5) to the park on my new bike yesterday. I __________ (6) to ride around the park, and my dad __________ (7) to walk with me.46.We __________ (go) to a concert last Saturday. It __________ (be) a wonderful experience. The band __________ (play) many of our favorite songs, and everyone__________ (enjoy) the music.47.Which animal can swim?A. DogB. CatC. FishD. Bird48.My school is very big. It __________ (1) a library, a playground, and a big gym. Every day, I __________ (2) to school by bus. The bus __________ (3) at 8:00 AM, and we __________ (4) to school together. After we __________ (5) school, I usually__________ (6) my homework at home. My friends and I __________ (7) often play basketball in the afternoon after school.49.Mia loves reading books. She has a shelf full of __________ (books) in her room. Today, she is reading a book about __________ (animals). The book has pictures of__________ (lions), __________ (elephants), and __________ (giraffes). Mia is very interested in learning about these __________ (animals).50.Which animal can fly?A. FishB. BirdC. LionD. Dog(答案及解释)。

人教PEP版六年级英语上册期中综合测试卷含答案

人教PEP版六年级英语上册期中综合测试卷含答案

人教PEP版六年级英语上册期中综合测试卷时间:60分钟满分:100分听力部分(30分)一、听句子,选出你所听到的单词或词组。

(5分)() 1. A. restaurant B. bookstore C. school() 2. A. by subway B. by plane C. on foot() 3. A. see a film B. take a trip C. draw pictures () 4. A. behind B. next to C. in front of() 5. A. slow B. stop C. wait二、听句子,给下列图片排序。

(5分)() ()()()() 三、听对话,选择正确的答案。

(10分)()1. Where is Amy going?A. She is going to the post office.B. She is going to the science museum. ()2. How can Jack get to the Chinese restaurant?A. Turn left at the cinema.B. Turn right at the cinema.()3. How does Mike often go to the park?A. By bike.B. On foot.()4. What is Lily going to do after school?A. She is going to see a film.B. She is going to visit her grandparents.()5. What is Tom going to buy this evening?A. He is going to buy a storybook.B. He is going to buy a dictionary.四、听对话,补全信息。

小学六年级上册第十三次英语第二单元期中试卷(答案和解释)

小学六年级上册第十三次英语第二单元期中试卷(答案和解释)

小学六年级上册英语第二单元期中试卷(答案和解释)英语试题一、综合题(本题有50小题,每小题2分,共100分.每小题不选、错误,均不给分)1.Lily is having a sleepover at her friend Emma’s house. They stay up late watching__________ and eating __________. They also play a fun game of __________, where they try t o guess each other’s favorite colors. Emma’s mom makes them __________ for breakfast, and they all enjoy it very much. After breakfast, Lily and Emma plan to go to the __________ to play with their other friends.2.Which of these is a common fruit?A. OrangeB. CarrotC. LettuceD. Potato3.What is the opposite of hot?A. ColdB. WarmC. BigD. Heavy4.What is the opposite of "old"?A. YoungB. SmallC. FastD. Big5.Which one is a time of day?A. MorningB. DogC. SpoonD. Car6.Sarah and Tom __________ (go) to the park last weekend. They __________ (play) football with their friends and __________ (have) a great time. After playing, they__________ (sit) under a tree and __________ (talk) about their plans for the summer vacation. Sarah __________ (suggest) that they __________ (visit) the beach next month. Tom __________ (agree) with her, saying it __________ (be) a wonderful idea.7.Which of these is used to eat food?A. KnifeB. SpoonC. PlateD. All of the above8.Which of these is an insect?A. DogB. CatC. ButterflyD. Elephant9.I like __________ books. My favorite book is about a __________ named__________ who goes on an __________. Tom is very __________, and he helps people in __________. The story takes place in a small __________ near a __________. Tom and his __________ explore the __________ and find many __________ things. There are also some __________ moments, but Tom never gives __________. I like this book because it is __________ and I enjoy reading about Tom’s __________.10.We __________ (go) to the zoo last Sunday. We __________ (see) many animals, like lions, tigers, and monkeys. The weather __________ (be) sunny and warm, and we __________ (have) a wonderful time.11.Which of these is a type of flower?A. PineB. RoseC. OakD. Maple12.Which of these animals can run fast?A. ElephantB. CheetahC. TurtleD. Dog13.Every Sunday, I __________ (help) my parents clean the house. I __________ (sweep) the floor, and my brother __________ (wash) the windows. We __________ (work) together and __________ (finish) everything quickly.14.I am going to visit my __________ (1) this weekend. They live in a small__________ (2) near a big __________ (3). I like to walk along the __________ (4) when I visit. Sometimes, I see __________ (5) flying in the sky. My grandmother loves __________ (6), so I help her __________ (7) the garden. Afterward, we sit together and have __________ (8).15.Which one is a famous tower?A. Eiffel TowerB. Big BenC. Leaning Tower of PisaD. All of the above16.Which of these is used to cook food?A. OvenB. SpoonC. PlateD. Chair17.Which one is a number?A. DogB. FiveC. ChairD. Spoon18.Yesterday, I __________ (go) to the zoo with my family. We __________ (see) many animals, like lions, tigers, and monkeys. The monkeys __________ (play) in the trees, and the lions __________ (sleep) in the sun. We __________ (take) pictures of the animals, and my little brother __________ (laugh) when he __________ (see) a funnymonkey. After that, w e __________ (eat) lunch at the zoo’s restaurant. It __________ (be)a wonderful day.19.Which of these is not a shape?A. SquareB. CircleC. TriangleD. Dog20.We _______ (have) a picnic last Sunday. We _______ (eat) sandwiches and_______ (drink) juice. The weather _______ (be) perfect for a picnic.21.What do we use to drink?A. SpoonB. CupC. PlateD. Fork22.Lucy is walking in the park. She sees some __________ (1) on the grass. Lucy picks a __________ (2) and puts it in her __________ (3). She loves flowers and often makes __________ (4) with them. After a while, she sits on a __________ (5) and enjoys the __________ (6).23.Next month, I ______ (visit) my grandparents. They ______ (live) in the countryside, so we ______ (go) hiking and ______ (pick) fruit from their garden.24.This morning, I __________ (ride) my bike to school. The weather __________ (be) sunny and warm. I __________ (arrive) at school early, so I __________ (have) some time to chat with my friends before class. We __________ (talk) about our weekend plans.25.I __________ (1) my homework in the afternoon. After I __________ (2) it, I__________ (3) to the park to play with my friends. We __________ (4) soccer and__________ (5) a good time. I __________ (6) to do it again next week.26.Which sentence uses a plural noun correctly?A. There are many book on the shelf.B. There are many books on the shelf.C. There are many bookes on the shelf.D. There are many bookes on the shelfs.27.Yesterday, my family and I went to a park for a picnic. We brought sandwiches, fruits, and drinks. After eating, we played frisbee and flew a kite. The weather was sunny, and everyone had a great time.28.What do we use to drink?A. KnifeB. ForkC. PlateD. Glass29.Which one is a fruit?A. TomatoB. PotatoC. CarrotD. Lettuce30.I like to __________ in my free time. My favorite activity is __________. I go to the __________ with my __________ to practice. I love to __________ the ball and run around the __________. It makes me feel very __________. Sometimes, we also play__________ with our classmates after the game.31.Which of these is a transportation?A. BicycleB. SpoonC. ChairD. Book32.We __________ (have) a picnic last weekend. We __________ (bring) sandwiches, fruit, and juice. It __________ (be) a beautiful day, and we __________ (sit) on the grass. We __________ (play) frisbee and __________ (take) pictures. We __________ (enjoy) the picnic very much.33.My grandparents __________ (live) in the countryside. They __________ (have) a big garden with many flowers. In the spring, the flowers __________ (bloom), and in the summer, they __________ (grow) vegetables. I __________ (visit) them every summer and __________ (help) them in the garden.34.Which of these is a color?A. GreenB. TableC. DogD. Chair35.Which one is used to eat food?A. ForkB. KnifeC. SpoonD. All of the above36.Which one is a pet?A. CowB. DogC. LionD. Elephant37.Which one is a number?A. ThreeB. PlateC. SpoonD. Knife38.Which sentence uses a preposition correctly?A. The book is on the table.B. The book on is the table.C. The table is on the book.D. The book is the on table.st Saturday, I __________ (1) to the park with my friends. It __________ (2) a sunny day, so we __________ (3) a picnic on the grass. We __________ (4) sandwiches, fruits, and juice. After we __________ (5) our lunch, we __________ (6) to play soccer. We __________ (7) a lot of fun. In the evening, we __________ (8) ice cream before going home. It __________ (9) a great day.40.My family loves going to the ______ during the summer. We swim in the ______ and build sandcastles. My sister likes to collect ______ from the shore, while I prefer tosearch for ______. After swimming, we often have a picnic with sandwiches, ______, and some ______. It’s a wonderful way to spend the day!41.What is the color of the sky on a sunny day?A. GreenB. BlueC. YellowD. Red42.Which of these is used to keep food cold?A. OvenB. FridgeC. MicrowaveD. Stove43.Which animal barks?A. CatB. DogC. BirdD. Cow44.This morning, I ______ (forget) to bring my schoolbag to school. I ______ (feel) embarrassed when I ______ (realize) it. My friend ______ (lend) me her extra pencil case, and I ______ (thank) her for it. I ______ (promise) to be more careful next time.45.Which one is a body part?A. ChairB. LegC. BookD. Car46.Which of these is a fruit?A. AppleB. LettuceC. CarrotD. Cucumber47.I __________ (go) to the library every Monday after school. My favorite book__________ (be) about adventure stories. I __________ (like) reading books because they __________ (take) me to different places. Yesterday, I __________ (borrow) a new book, and I __________ (start) reading it right away.48.I __________ (usually/get) up at 7:00 a.m. But today I __________ (sleep) in because it __________ (be) a holiday. I __________ (wake) up at 9:00 a.m. and__________ (feel) very relaxed.49.What is the opposite of "clean"?A. DirtyB. WetC. SmallD. Big50.We went to the zoo last weekend and saw many animals. My favorite animal was the __. It was so big and strong. We also saw a cute little __ playing with its friends. After that, we visited the __ and watched them swim. It was a fun day at the zoo!(答案及解释)。

北京版英语小学六年级上学期期中试题及解答参考(2024年)

北京版英语小学六年级上学期期中试题及解答参考(2024年)

2024年北京版英语小学六年级上学期期中模拟试题及解答参考一、听力部分(本大题有12小题,每小题2分,共24分)1、Listen to the following dialogue and choose the best answer to complete the sentence.A. What’s the weather like today?B. How was your weekend?C. Did you go to the park yesterday?Answer: BExplanation: The speaker is asking about the listener’s weekend, indicating a casual and friendly conversation. Therefore, the best answer isB.2、Listen to the following passage and answer the question.Question: What are the students doing in this passage?A. They are having a science experiment.B. They are watching a movie.C. They are playing a game.Answer: AExplanation: The passage mentions that the students are conducting anexperiment in science class, so the correct answer is A.3、Question: What are the students doing in the morning break?A. They are playing soccer on the playground.B. They are having a class in the classroom.C. They are reading books in the library.Answer: AExplanation: The students are described as playing soccer on the playground in the conversation, so the correct answer is A.4、Question: How many students are in the school’s basketball team?A. 8B. 10C. 12Answer: BExplanation: The teacher mentions that there are ten students in the school’s basketball team, so the cor rect answer is B.5.You are listening to a conversation between two students, Alice and Bob, discussing their weekend plans. Listen carefully and answer the following question.Question: What activity do Alice and Bob plan to do together this weekend?A. Go shoppingB. Watch a movieC. Go hikingD. Visit a museumAnswer: CExplanation: The conversation goes like this: “Alice: Hi Bob, do you have any plans for the weekend?” “Bob: No, not really. How about you?” “Alice: I was thinking of going hiking. Wou ld you like to join me?” “Bob: Sure, that sounds fun!”6.Listen to a short dialogue between a teacher and a student named Lily about her science project. Then answer the following question.Question: What is Lily’s science project about?A. WeatherB. PlantsC. AnimalsD. The solar systemAnswer: BExplanation: The dialogue goes like this: “Teacher: Lily, how is your science project coming along?” “Lily: It’s going well. I’m doing a project on different types of plants.” “Teacher: That’s great! Plants are f ascinating subjects to study.”7、Listen to the conversation and choose the correct answer.A. The boy is reading a book.B. The girl is playing the piano.C. They are doing their homework together.Answer: CExplanation: In the conversation, the boy says, “Let’s do our homework together.” and the girl agrees. Therefore, the correct answer is C.8、Listen to the passage and answer the question.Question: What is the weather like today?A. SunnyB. RainyC. CloudyAnswer: BExplanation: In the passage, the speaker says, “The weather forecast says it will be rainy today.” Therefore, the correct answer is B.9、What does the boy want to do after school?A) Go swimming.B) Play basketball.C) Visit the library.Answer: B) Play basketball.Explanation: In the dialogue, the boy mentions that he plans to go to the park to play basketball with his friends right after school.10、How does the girl feel about the upcoming field trip?A) Excited.B) Worried.C) Indifferent.Answer: A) Excited.Explanation: When asked about the field trip next week, the girl responds with enthusiasm, saying she can’t wait to visit the science museum and see all the exhibits.11.Listen to the conversation and answer the question.A. What is the weather like today?B. Where is Tom going for the weekend?C. How is Tom feeling this morning?Answer: BExplanation: The conversation mentions, “Tom, are you ready for the weekend trip?” indicating that the topic is about where Tom is going for the weekend.12.Listen to the dialogue and complete the sentence with the correct word.The teacher asked the students to bring__________to the school assembly.A. a dictionaryB. their booksC. the slidesAnswer: BExplanation: The teacher’s request for the students to bring something to the school assembly is likely related to a presentation or activity, making “their books” a reasonable answer as they often carry books to school events. The other options, a dictionary and the slides, are less likely as they are not typically items brought to a school assembly unless specified.二、选择题(本大题有12小题,每小题2分,共24分)1、Choose the correct form of the verb to complete the sentence:•The students (study, studies) very hard every day.•Answer: studies•Explanation: In the third person singular present tense, the verb usually ends with “-s”. Here, “The students” is considered a singular entity performing the action, so we use “studies”.2、Select the correct preposition to fill in the blank:•We arrived________Beijing at 9 o’clock in the morning.•Options: A) at, B) in, C) on•Answer: B) in•Explanation: When talking about arriving at a large place such as a city, we use the preposition “in”. Thus, “We arrived in Beijing” is the correct choice.3、What is the capital city of China?A. ShanghaiB. BeijingC. GuangzhouD. Hong KongAnswer: BExplanation: Beijing is the capital city of China. While Shanghai, Guangzhou, and Hong Kong are all significant cities in China, they are not the capital.Shanghai is known for its economic importance, Guangzhou is famous for its history and culture, and Hong Kong is a special administrative region known for its financial and trade sectors.4、Which of the following is a famous historical site in Beijing?A. The Great Wall of ChinaB. The Eiffel TowerC. The Statue of LibertyD. The ParthenonAnswer: AExplanation: The Great Wall of China is a famous historical site located in Beijing. The Eiffel Tower is located in Paris, France; the Statue of Liberty is located in New York City, United States; and the Parthenon is located in Athens, Greece. These are iconic landmarks in their respective countries but are not located in Beijing.5、Which sentence correctly uses the past perfect tense?A) She had finished her homework before she went to bed.B) She finished her homework before she goes to bed.C) She has finished her homework before she went to bed.D) She finish her homework before she went to bed.Answer: AExplanation: The correct choice is A because it uses the past perfect tense (“had finished”) to indicate that one action (finishing homework) occurredbefore another action (going to bed) in the past. Choices B, C, and D do not use the correct verb tenses or forms to convey this sequence of events.6、Choose the sentence that is grammatically correct:A) If I would have known, I wouldn’t have come.B) If I had known, I wouldn’t have come.C) If I know, I won’t come.D) If I knew, I won’t have come.Answer: BExplanation: The correct choice is B. This sentence properly uses the third conditional, which talks about a hypothetical situation in the past and its possible results. The structure “If + past perfect, would have + past participle” is used to describe a situation that did not happen and its imagined consequences. Choice A incorrectly mixes the second and third conditionals. Choice C uses the first conditional, which does not fit the context of a past hypothetical situation. Choice D incorrectly combines the second and third conditionals.7、What is the correct pronunciation of the word “shelter” in American English?A. /ˈʃɛltər/B. /ˈʃɛltər/C. /ˈʃɛltɚ/D. /ˈʃɛltɚ/Answer: B. /ˈʃɛltər/Explanation: The correct pronunciation of “shelter” in American English is /ˈʃɛltər/. The ‘e’ in “shelter” is long, as in the word “see.”8、Which of the following sentences is grammatically correct?A. The children has gone to the park.B. The children have gone to the park.C. The children has been gone to the park.D. The children has been gone to the park.Answer: B. The children have gone to the park.Explanation: The correct sentence is B because “children” is a plural noun, and the correct form of the verb “to go” for a plural subject is “have gone.” The other options are incorrect due to incorrect verb agreement and tense usage.9、Whic h of the following sentences uses the correct form of the verb ‘to be’?A)She are a good student.B)He am from China.C)They is very happy.D)I am learning English.Answer: D) I am learning English.Explanation:The verb ‘to be’ changes its form based on the subject. ‘I’ is followed by ‘am’, ‘he/she/it’ is followed by ‘is’, and ‘you/we/they’ is followed by ‘are’. In option D, ‘I’ is correctly matched with ‘am’.10、Choose the sentence that has the same meaning as: “It’s not far fro m here to the park.”A)The park is close to here.B)Here is a long way from the park.C)The distance from the park to here is big.D)It’s a short distance between the park and there.Answer: A) The park is close to here.Explanation: The original sentence means that the park can be reached without traveling a great distance. Option A conveys this meaning by stating that the park is near or at a short distance from the current location.11、The following sentence contains an error in punctuation. Which one is it?A. “She went to the park, and bought some apples, and then went home.”B. “She went to the park and bought some apples, then went home.”C. “She went to the park; bought some apples; and then went home.”D. “She went to the park, bought some apples, then went home.”Answer: CExplanation: The correct punctuation should be used to separate the independent clauses. In option C, there should not be a comma after “park” and “apples” as they are not independent clauses but part of the main clause. The correct sente nce should be “She went to the park; bought some apples; then went home.”12、Choose the correct word to complete the sentence.The teacher asked the students to write a story about a (circle the correct word) adventure.A. excitingB. excitedC. exciteD. excitinglyAnswer: AExplanation: The correct word to complete the sentence should be an adjective that describes the type of adventure. “Exciting” is the correct choice because it is an adjective that means something that is thrilling or enjoyable. The other options are either verbs or adverbs, which do not fit the context of the sentence.三、完型填空(10分)Beijing Edition English, Grade 6, First Semester Midterm ExamPart III: Cloze Test (10 points)Read the passage below and choose the best answer to complete the sentences.It was a bright and sunny day. Sarah and her family decided to go on a picnic in the park. They packed some delicious food and drinks. When they arrived at the park, they found a nice spot under a big tree. The tree provided(1)_______shade for them to enjoy their meal. After eating, Sarah’s brother, Tom, suggested they play (2) _______. Everyone agreed, and they had a lot of fun playing together. Later, Sarah wanted to read a book, so she took out her favorite storybook from her backpack. She sat down and started reading, whileher parents were chatting and enjoying the (3) _______. Suddenly, a little squirrel came close to them, looking for some (4) _______. Sarah gave it a small piece of bread, and the squirrel happily accepted it. It was such a (5)_______day, and Sarah felt very happy that they spent time as a family.1.__________2.__________3.__________4.__________5.__________A. coolB. warmC. hotA. soccerB. chessC. cardsA. weatherB. picnicC. sceneryA. waterB. foodC. shelterA. boringB. tiringC. wonderfulAnswers:1.A. cool2.A. soccer3.C. scenery4.B. food5.C. wonderfulThis cloze test is designed to be suitable for a 6th-grade level, with vocabulary and context that should be familiar and engaging for students.四、阅读理解(26分)Reading ComprehensionPassage:In the heart of Beijing, the Forbidden City stands as a testament to the grandeur of ancient Chinese emperors. Covering an area of 720,000 square meters, the Forbidden City was the imperial palace for 24 emperors during the Ming and Qing dynasties. It was built in the early 15th century and took 14 years to complete. The palace complex consists of 96 palaces and courtyards, with 8707 rooms in total. The walls of the Forbidden City are 10 meters high and 860 meters long, encircling the entire palace grounds.The Forbidden City was not only a political center but also a spiritual andcultural hub. Emperors would hold important ceremonies, such as the Imperial Sacrifice, to show their respect for ancestors and gods. The architecture of the Forbidden City is a blend of Chinese traditional style and Ming and Qing dynasty characteristics. The main buildings, such as the Hall of Supreme Harmony, the Hall of Middle Harmony, and the Hall of Preserving Harmony, are grand and imposing, showcasing the power and majesty of the imperial family.Today, the Forbidden City is a major tourist attraction in Beijing. It offers a glimpse into the past and a chance to appreciate the rich cultural heritage of China. Many visitors are fascinated by the intricate carvings, paintings, and calligraphy found throughout the palace.Questions:1.How long did it take to build the Forbidden City?A) 10 yearsB) 14 yearsC) 20 yearsD) 24 years2.What is the total number of palaces and courtyards in the Forbidden City?A) 96B) 192C) 8707D) 10,0003.What is the main purpose of the Imperial Sacrifice?A) To c elebrate the emperors’ birthdaysB) To honor ancestors and godsC) To display the emperors’ wealthD) To practice martial artsAnswers:1.B) 14 years2.A) 963.B) To honor ancestors and gods五、写作题(16分)Writing Prompt (题目):Imagine you are writing a letter to your pen pal in the United Kingdom. Your pen pal wants to know about your favorite traditional Chinese festival. Choose one festival that you enjoy the most and describe it in detail. Explain why this festival is important to you and your family, what special activities or customs are associated with it, and any personal experiences you have had during this celebration. Remember to use descriptive language to help your pen pal understand the significance of the festival.Sample Response (写作示例):Dear [Pen Pal’s Name],I hope this letter finds you well. I was delighted to receive your last letter and learn more about the festivals celebrated in the UK. In return, I would like to share with you my favorite traditional Chinese festival, which is theMid-Autumn Festival, also known as the Moon Festival.The Mid-Autumn Festival falls on the 15th day of the 8th lunar month, when the moon is believed to be at its fullest and brightest. It is a time when families gather together to appreciate the beauty of the full moon, eat delicious mooncakes, and light colorful lanterns. The festival symbolizes unity and happiness, and it is one of the most important celebrations in Chinese culture.In my family, we usually start preparing for the festival a few days before it arrives. My grandmother makes homemade mooncakes with lotus seed paste and salted egg yolks, which are traditional fillings. The process is quite intricate, but the result is worth the effort. On the night of the festival, we go to a nearby park where many people are out enjoying the moonlight. We bring our own lanterns and join in the fun. There are dragon dances and performances, which add to the festive atmosphere.One of my fondest memories of the Mid-Autumn Festival was last year when my cousin and I made our very own lanterns using bamboo sticks and colored paper. It was challenging, but we were so proud when we lit them up and joined the parade. The festival not only brings joy but also teaches us the value of family and tradition.I hope this gives you a glimpse into how we celebrate the Mid-Autumn Festival in China. Perhaps next year, if you have the chance to visit China during this time, you can experience it firsthand!Best wishes,[Your Name]Analysis (解析):This letter effectively communicates the significance of the Mid-Autumn Festival to a foreign audience by providing a detailed description of the event, including its date and traditions. The writer successfully incorporates personal experiences to make the festival come alive, making the letter engaging and informative. The choice of vocabulary and descriptive language helps the reader visualize the celebrations and understand their importance to the writer and their family. This approach not only fulfills the task requirements but also encourages cultural exchange and understanding between the writer and their pen pal.This writing prompt and sample response should provide a good framework for evaluating the student’s ability to express themselves in Engli sh while sharing aspects of their culture.。

人教版英语小学六年级上学期期中试题及答案指导(2024年)

人教版英语小学六年级上学期期中试题及答案指导(2024年)

2024年人教版英语小学六年级上学期期中复习试题及答案指导一、听力部分(本大题有12小题,每小题2分,共24分)1、听力原文:W: Good morning, Class 6! Can you tell me your name, please?M: Hello, Miss. I’m Li Ming.Question: What is the student’s name?A) Li MingB) Li MeiC) Li HongD) Li JianAnswer: A) Li MingExplanation: The student introduces himself as “Li Ming,” which is clearly stated in the dialogue. Therefore, the correct answer is A) Li Ming.2、听力原文:W: Excuse me, are you going to the library after school?M: Yes, I am. I need to borrow some books for my science project.Question: Why is the boy going to the library?A) To borrow books for a science projectB) To meet his friendsC) To study for a math testD) To read a novelAnswer: A) To borrow books for a science projectExplanation: The boy explicitly states that he needs to borrow books for his science project. Therefore, the correct answer is A) To borrow books fora science project.3、Listen to the following dialogue and choose the best answer for each question.A. What is the weather like today?B. How do you go to school?C. What are you doing after school?D. How much is the book?Answer: CExplanation: The question asks about what the speaker is doing after school. The correct answer can be found in the dialogue where the speaker mentions, “After school, I am going to the library.”4、Listen to the passage about a famous landmark and answer the following question.What is the name of the famous landmark in Paris?A. The Tower of LondonB. The Eiffel TowerC. The ColosseumD. The Great WallAnswer: BExplanation: The passage clearly mentions that the famous landmark in Paris is the Eiffel Tower, making option B the correct answer. The other options are landmarks in other cities.5、Listen to the conversation and answer the question.W: Hi, Tom. How was your weekend?M: Oh, it was great! I went hiking with my family. We saw a lot of beautiful scenery and even spotted some wildlife.Question: What did Tom do over the weekend?A) He went to the beach.B) He went hiking.C) He stayed at home.Answer: BExplanation: The question asks what Tom did over the weekend. In the conversation, Tom mentions that he went hiking with his family, so the correct answer is B) He went hiking.6、Listen to the dialogue and fill in the blanks with the correct words. W: Hi, John. I heard you joined the school’s science club. That’s great! M: Yes, it is. I’m really excited about it. We’re going to have a special presentation next week.W: Oh, that sounds interesting. What will you be presenting?M: Well, we’re going to talk about the solar system. I’m responsible for explaining about the planets.W: That’s fascinating! I always wanted to learn more about space.M: You should definitely join us next week. It’s going to be a lot of fun.Question: What are they planning to present about?A) The history of space exploration.B) The solar system.C) The mysteries of the universe.Answer: BExplanation: The dialogue clearly states that they are going to talk about the solar system. Therefore, the correct answer is B) The solar system.7.Listen to the following conversation and choose the best answer.A. What is the weather like today?B. Where are they going for vacation?C. How are they planning to spend their weekend?Answer: BExplanation: In the conversation, the speakers are discussing their vacation plans, which indicates that the topic of the conversation is about where they are going for vacation.8.Listen to the following dialogue and answer the question.Question: What is the main reason the woman can’t attend the party?A. She has a cold.B. She has to work late.C. She is going on a trip.Answer: BExplanation: The woman explicitly mentions that she has to work late, which is the reason she cannot attend the party. This makes option B the correct answer.9.Listen to the following dialogue and choose the best answer to complete the sentence.A: Excuse me, where is the nearest post office?B:__________a.It’s on the left of the street.b.It’s on the right of the street.c.It’s across the street.d.It’s behind the bank.Answer: cExplanation: The question asks for the location of the nearest post office. The correct answer is “It’s across the street,” which is indicated by optionc.10.Listen to the following passage and answer the question.W: Hello, my name is Lisa. I’m from China. I’m here to study English.M: Hi, Lisa. It’s great to meet you. Where are you studying?W: I’m studying at XYZ English School.M: That’s a nice school. Do you like it?W: Yes, I do. The teachers are very nice, and the students are friendly.Question: Where is Lisa studying?a.ABC English Schoolb.XYZ English Schoolc.PQR English Schoold.STU English SchoolAnswer: bExplanation: The passage clearly states that Lisa is studying at XYZ English School, which is indicated by option b.11.You are listening to a conversation between two students, Tom and Lucy. Listen carefully and choose the correct answer.A. Tom is reading a book.B. Lucy is watching a movie.C. They are both studying English.Answer: BExplanation: The convers ation goes like this: “Hi, Tom, what are you doing?” “Hi, Lucy. I’m watching a movie. How about you?” “I’m reading a book. It’s really interesting.” The correct answer is B because Lucy mentions that sheis watching a movie.12.Listen to a short dialogue between a teacher and a student about the weekend plans. Then, choose the correct answer.A. The student is planning to go to the park with friends.B. The student is planning to visit a museum.C. The student is planning to stay at home and do some homework.Answer: AExplanation: The dialogue goes like this: “Teacher: What are you planning to do this weekend, Tom?” “Student: I’m planning to go to the park with my friends. We’re going to play soccer and have a picnic.” The correct answer is A because the student mentions that they are planning to go to the park with friends.二、选择题(本大题有12小题,每小题2分,共24分)1、Choose the correct word to complete the sentence.The cat is very __________, so it always keeps its fur clean.A. lazyB. cleverC. tidyD. hungryAnswer: C. tidyExplanation: The sentence is about the cat’s habit of keeping its fur clean, which suggests it is well-groomed and organized. “Tidy” is the correct choice as it implies cleanliness and orderliness.2、Select the word that is spelled differently from the others.A. changeB. flameC. strangeD. changeableAnswer: D. changeableExplanation: All the other words (change, flame, strange) have a double ‘e’ at the end. “Changeable” is the only word in the list that does not follow this pattern, making it the correct answer.3、Choose the correct word to complete the sentence.The teacher asked the students to turn off their__________before the class began.A. computersB. booksC. pensD. bagsAnswer: A. computersExplanation: The correct answer is “computers” because it is common for students to turn off their electronic devices, such as computers, before starting a class. The other options, while relevant to school, are not typically things that would need to be turned off before the class begins.4、Read the sentence and choose the best word to fill in the blank.She was__________surprised to see her friend after all these years.A. reallyB. veryC. muchD. soAnswer: D. soExplanation: The correct answer is “so” because it is used to emphasize the degree of surprise. The phrase “so surprised” is a common way to express a very high level of surprise. The other options can also express surprise, but they do not carry the same emphasis as “so.”5.Choose the correct word to complete the sentence.The cat is sleeping on the ____________.A. chairB. floorC. deskAnswer: BExplanation: The correct answer is “floor” because it is common to find a cat sleeping on the floor. “Chair” and “desk” are less likely places fora cat to sleep.6.Select the sentence that correctly uses the past tense.A. I go to the park yesterday.B. He was walking to the store last night.C. We are eating dinner at the restaurant now.Answer: BExplanation: The correct sentence is “He was walking to the store last night” because it uses the past continuous tense, indicating an action that was happening in the past. “I go to the park yesterday” is incorrect because “go” should be “went” in the past tense. “We are eating dinner at the restaurant now” is in the pre sent continuous tense and does not match the requirement of using the past tense.7、Choose the correct word to complete the sentence.The cat is very ______; it always lands on its feet.A)braveB)carefulC)luckyAnswer: C) luckyExplanation: The sentence implies that the cat has a remarkable ability to land on its feet, which is often associated with luck rather than bravery or caution.8、Select the sentence that does not follow the correct subject-verb agreement.A)The children are playing outside.B)Each child has their own book.C)She and her friends are going to the park.Answer: B) Each child has their own book.Explanation: In English, the correct subject-verb agreement for the phrase“each child” is “has” (singular verb), but the sentence incorrectly uses “their,” which is plural. The correct sentence should be “Each child has his or her own book.”9.Choose the correct word to complete the sentence:The cat was sleeping on the mat when the___________dog barked loudly.A. tinyB. angryC. happyD. lazyAnswer: BExplanation: The correct answer is “angry” because it implies that the dog’s barking was caused by frustration or irritation, which is a common reason fora dog to bark loudly.10.Select the sentence that has the correct subject-verb agreement.A. The students is studying in the library.B. Each student are reading books.C. The children has finished their homework.D. Both of the boys are playing outside.Answer: DExplanation: The correct answer is “Both of the boys are playing outside” because “both” is a pronoun that requires a plural verb, and “are” is the correct plural form of the verb “to be.” The other options have incorrectsubject-verb agreement.11.Choose the correct word to complete the sentence.The teacher was (A) angry, (B) surprised, (C) excited about the students’ performance in the play.Answer: BExplanation: The sentence is about the teacher’s reaction to the students’ performance. “Surprised” is the most appropriate word to describe thete acher’s reaction, as the performance may have been unexpected.12.Select the word that does not belong in the following list.(A)umbrella (B) car (C) bicycle (D) airplane (E) phoneAnswer: EExplanation: All the other items in the list are means of transportation that can be used on land. A phone is not a form of transportation and is used for communication purposes, making it the odd one out.三、完型填空(10分)Directions: There are 10 blanks in the following passage. For each blank, there are four choices marked A, B, C, and D. Please choose the best one for each blank.My favorite animal is the bear. Bears are very strong and intelligent. They live in the forest and eat a lot of different things. Bears can be black, brown, or even white. They are mostly herbivores, but some can also eat meat.One day, a bear was walking in the forest. He saw a small stream of water.He was very thirsty and wanted to drink. However, the stream was surrounded by high rocks. The bear tried to jump over the rocks, but he couldn’t make i t. He looked around and saw a vine growing near the rocks. He thought, “Maybe I can use this vine to climb over the rocks.”1.My favorite animal is the bear. It is a(an)________animal.A. strong and intelligentB. weak and lazyC. cute and playfulD. fierce and dangerous2.Bears live in the ________.A. cityB. forestC. oceanD. desert3.Bears can be ________.A. only blackB. only brownC. only whiteD. black, brown, or white4.Bears are mostly ________.A. carnivoresB. herbivoresC. omnivoresD. birds5.The bear saw a________growing near the rocks.A. treeB. flowerC. vineD. rockAnswers:1.A2.B3.D4.B5.C四、阅读理解(26分)Reading ComprehensionRead the following passage and answer the questions that follow.In the small town of Willow Creek, there was once a magnificent old oak tree that stood tall and proud at the center of the town square. The tree was known to everyone as “Old oak.” It was not just a tree; it was a symbol of the town’s history and community spirit. The children would gather under its broad branches during summer afternoons, the elders would sit on its roots and share stories,and during festive occasions, it was the perfect backdrop for parades and concerts.One day, a storm came, and the old oak tree was struck by lightning. Its branches snapped and its trunk began to crack. The townspeople were heartbroken at the sight of their beloved tree. They decided to take action and raise funds to save the old oak. The whole community contributed, from the smallest child to the oldest resident, and soon enough, enough money was gathered to hire a team of experts to help.The team worked tirelessly to stabilize the tree. They reinforced the trunk with steel and replaced the snapped branches with new, healthy ones. The old oak tree was saved, and the community celebrated with a grand festival. From that day on, they took even better care of the tree, ensuring it would stand tall for generations to come.Questions:1.Why was the old oak tree in Willow Creek significant to the community?A) It was the town’s only tree.B) It was a symbol of the town’s history and community spirit.C) It was the home of a local squirrel family.D) It provided shade for the town’s animals.2.What happened to the old oak tree during the storm?A) It was completely destroyed.B) Its branches snapped and its trunk cracked.C) It fell over due to its age.D) It was uprooted by the wind.3.How did the community help save the old oak tree?A) They planted new trees around it.B) They offered prayers for its recovery.C) They raised funds and hired experts to repair it.D) They moved the tree to a safer location.Answers:1.B) It was a symbol of the town’s history and community spirit.2.B) Its branches snapped and its trunk cracked.3.C) They raised funds and hired experts to repair it.五、写作题(16分)Task: Write a short story about a day you remember well. Your story should include the following elements:•The date and the occasion of the event.•The setting and the atmosphere of the day.•Three key events that happened, and their impact on you.•How the day has affected you since then.Example:Title: The Day of the FairOn October 15th, 2021, I experienced one of the most memorable days of my life. It was the annual school fair, and the whole campus was buzzing withexcitement.Setting and Atmosphere:The morning was crisp, with a gentle breeze that carried the scent of autumn leaves. The fairground was decorated with colorful balloons and festive banners. The atmosphere was electric, with children laughing and parents cheering.Key Events:1.Winning the Raffle: I had entered the raffle with a small toy car, and to my surprise, my name was drawn! I was so thrilled that I could barely contain my excitement.2.Performing in the Talent Show: Later in the afternoon, I decided to participate in the talent show. I sang a song I had been practicing for weeks, and the crowd cheered me on. I felt a sense of pride and accomplishment that day.3.Visiting the Craft Booth: I spent some time at the craft booth, makinga friendship bracelet with a fellow classmate. It was a simple activity, but it made me realize the importance of friendship and sharing.Impact and Reflection:Since that day, I have carried the joy and sense of community with me. The fair taught me the value of hard work, the importance of having fun, and the joy of making memories with friends. Every time I look at the friendship bracelet I made, I am reminded of that wonderful day and the positive impact it had on my life.Analysis:This example follows the structure outlined in the task. It starts with a clearintroduction of the date and occasion, followed by a description of the setting and atmosphere. The three key events are detailed, showing their impact on the narrator. Finally, the story concludes with a reflection on how the day has affected the narrator’s life since then, providing a personal touch and closure to the story.。

新课标人教版小学英语六年级上册期中试卷含参考答案

新课标人教版小学英语六年级上册期中试卷含参考答案

专业课原理概述部分一、选择题(每题1分,共5分)1. 英语单词"cat"的复数形式是______。

A. catsB. catesC. catessD. caties2. 下列哪个单词的发音与其他三个不同?A. carB. cakeC. cardD. case3. "He ___________ to the park every Sunday."(他每个星期天都去公园。

)A. goB. goesC. goingD. to go4. "I have two __________."(我有两个兄弟。

)A. brotherB. brothersC. sisterD. sisters5. "She ___________ a teacher."(她是一名老师。

)A. amB. isC. areD. be二、判断题(每题1分,共5分)1. "Apples"是"apple"的复数形式。

()2. "I like cats"意思是"我喜欢狗"。

()3. "He is reading a book"表示"他在读书"。

()4. "She has a pen"意思是"她有一支钢笔"。

()5. "They are students"表示"他们是老师"。

()三、填空题(每题1分,共5分)1. My mother is a ________. She works in a hospital.2. I have ________ apples. One is red and the other is green.3. ________ you like tea or coffee?4. They ________ to the movies last night.5. There are ________ days in a week.四、简答题(每题2分,共10分)1. 请写出"狗"的英文单词。

牛津译林版英语小学六年级上学期期中试题及解答参考(2024-2025学年)

牛津译林版英语小学六年级上学期期中试题及解答参考(2024-2025学年)

2024-2025学年牛津译林版英语小学六年级上学期期中自测试题及解答参考一、听力部分(本大题有12小题,每小题2分,共24分)1、What does the boy want for breakfast?A) Toast and juiceB) Cereal and milkC) Eggs and baconAnswer: B) Cereal and milkExplanation: In the dialogue, the boy asks his mother if there’s any cereal left because he forgot to eat it yesterday. His mother confirms that there is still some cereal and fresh milk in the fridge.2、Where did the girl lose her backpack?A) At the libraryB) On the school busC) In the parkAnswer: A) At the libraryExplanation: During the conversation, the girl mentions that she realized her backpack was missing after she returned from studying at the library. She thought she had placed it under the table there.3、Listen to the dialogue between two friends and answer the question.A. What are the friends doing?B. Where are they going?C. What time is it?Answer: B. Where are they going?Explanation: Listen carefully to the dialogue. You will hear one of the friends say, “Let’s go to the library after school.” This indicates that they are discussing where to go after school.4、Listen to the passage about animals and choose the correct answer to the question.What is the main idea of the passage?A. The passage is about the different types of animals in the forest.B. The passage talks about how animals communicate with each other.C. The passage describes the habitat of various animals.Answer: C. The passage describes the habitat of various animals.Explanation: Listen to the passage carefully. The speaker talks about different animals and their living environments, such as forests, mountains, and oceans. The main idea of the passage is to describe the habitats of various animals.5、Listen to the conversation and choose the correct answer. (Listen once)A. They are going to the cinema.B. They are planning a picnic.C. They are discussing homework.Answer: B. They are planning a picnic.Explanation: In the conversation, you hear two friends talking about buying food and drinks and checking the weather forecast for tomorrow. This indicates they are making plans for a picnic.6、Listen to the short passage and decide if the following statement is true or false. (Listen twice)The school’s annual sports day wil l be held next Friday.Answer: True.Explanation: The passage mentions that all students should wear their sports uniforms and gather on the playground by 8 AM next Friday for the start of the school’s annual sports day event.7.Listen to the conversation between two students, Tom and Alice, discussing their weekend plans. Choose the correct answer from the options A, B, C, or D.A. Tom and Alice are going to the beach this weekend.B. Tom and Alice are planning to stay home and watch movies.C. Tom and Alice are going to visit their grandparents.D. Tom and Alice are preparing for a science fair.Answer: CExplanation: In the conversation, Tom mentions that they are visiting their grandparents in the countryside, which indicates option C is the correct answer.8.Listen to a short passage about a famous author, J.K. Rowling. Answer the following question based on the passage.Question: What inspired J.K. Rowling to write the Harry Potter series?A. Her experience of living in a small village.B. Her love for animals and nature.C. Her own childhood memories.D. Her fascination with the supernatural.Answer: CExplanation: In the passage, it is mentioned that J.K. Rowling drew inspiration from her own childhood memories to create the magical world of Harry Potter, making option C the correct answer.9、Listen to the dialogue and choose the correct answer.Dialogue:M: Excuse me, could you tell me where the nearest bookstore is?W: Of course. You need to go straight down this street until you reach the traffic lights. Turn left there, and the bookstore is on your right, next to the coffee shop.Question:Where is the bookstore according to the woman?A. Next to the post office.B. On the left of the coffee shop.C. On the right of the coffee shop.Answer: C. On the right of the coffee shop.Explanation: The woman in the dialogue clearly states that after turningleft at the traffic lights, the bookstore is on the right side, next to the coffee shop.10、Listen to the conversation and choose the correct answer.Dialogue:W: What do you think about the new science teacher?M: Well, his classes are always full of interesting experiments, but sometimes I wish he would slow down a bit because he covers too much material too quickly.Question:What does the bo y think about the new science teacher’s classes?A. They are too boring.B. They are too fast-paced.C. They are too slow.Answer: B. They are too fast-paced.Explanation: The boy mentions that while the classes are interesting due to the experiments, he feels that the teacher covers too much material too quickly, indicating that the pace of the class is too fast for his liking.11.Listen to the conversation between two students about their weekend plans. Choose the best answer.A. They are going to the beach.B. They are planning to visit their grandparents.C. They are going to a movie.Answer: BExplanation: The conversation mentions that they haven’t seen their grandparents for a while and are planning to visit them this weekend, which indicates that option B is correct.12.Listen to the following dialogue and answer the question.Why does the bo y want to borrow the girl’s bike?A. He has a flat tire.B. His mother forgot to buy him a new bike.C. He wants to ride it to school because it’s sunny.Answer: CExplanation: The boy says, “I really need your bike today because it’s such a nice day outsi de. I want to ride it to school instead of walking.” This indicates that option C is the correct answer.二、选择题(本大题有12小题,每小题2分,共24分)1、Which of the following words has a different pronunciation for the underlined part?A)heavyB)horseC)hourD)houseAnswer: C) hourExplanation:In options A, B, and D, the ‘h’ at the beginning of the word is pronounced, but in “hour,” the ‘h’ is silent, making its pronunciationdifferent from the other choices.2、What does the phrase “break the ice” mean?A)To start a conversation in a friendly wayB)To interrupt someone when they are speakingC)To end a meeting or gatheringD)To solve a difficult problemAnswer: A) To start a conversation in a friendly wayExplanation:The phrase “break the ice” is used to describe the action of doing or saying something to make people feel more relaxed and comfortable, especially when they meet for the first time or if there’s an awkward silence.3、What is the correct plural form of the word “child”?A. childsB. childrenC. childesD. childenAnswer: B. childrenExplanation: The plural form of “child” is “children.” The other options are incorrect spellings.4、Which of the following sentences uses the past continuous tense correctly?A. He was walking to the park when it started to rain.B. I am eating ice cream right now.C. She has been sleeping for 8 hours.D. They went to the movies yesterday.Answer: A. He was walking to the park when it started to rain.Explanation: The past continuous tense is used to describe an action that was ongoing at a specific time in the past. Option A correctly uses the past continuous tense (“was walking”) to indicate an action that was happening when another past action occurred (“started to rain”). Options B and D use the simple past tense, and option C uses the present perfect continuous tense.5、Choose the correct word to complete the sentence.The cat was___________on the windowsill, looking out at the birds.A)layingB)lyingC)lying downD)layAnswer: B) lyingExplanation: The correct word here is “lying,” which means to be in a horizontal position, often relaxed. The cat is simply in a horizontal position on the windowsill, not sleeping or doing anything else. “Laying” (A) is incorrect because it implies the act of placing something down, and “lying down”(C) is too specific as it suggests the cat is in a resting position, which is not necessary to describe the action of simply being horizontal. “Lay” (D)is the past tense of “lie” and does not fit the present continuous context of the sentence.6、Complete the sentence with the correct form of the verb in parentheses. The children___________(be) very excited about the upcoming school trip.A)areB)isC)wereD)beAnswer: A) areExplanation: The correct form of the verb is “are,” as it matches the plural subject “The children.” The sentence describes a present state of excitement, so the present simple tense is appropriate. “Is” (B) is incorrect because it is the singular form of the verb and does not match the plural subject. “Were” (C) is the past tense and does not fit the present context. “Be” (D) is the base form of the verb and does not agree with the plural subject in terms of number.7、Choose the correct word to complete the sentence.The cat is_______(sleeping/awake) on the mat.A. sleepingB. awakeAnswer: AExplanation: The sentence talks about a cat on a mat, which implies it is resting. Therefore, “sleeping” is the correct word to complete the sentence.8、Select the word that is different from the others in the group.A. umbrellaB. appleC. pencilD. watchAnswer: BExplanation: The words “umbrella,” “pencil,” and “watch” are all objects that people use. “Apple,” on the other hand, is a type of fruit, which makes it different from the other words in the group.9.Choose the word that best completes the sentence.The teacher praised the students for their__________behavior in the classroom.A)aggressiveB)cooperativeC)passiveD)rebelliousAnswer: B) cooperativeExplanation: The correct answer is “cooperative” because it implies that the students were working well together and showing a willingness to help and collaborate with others, which is positive behavior in a classroom setting. The other options (“aggressive,” “passive,” “rebellious”) suggest behavior that is not typically praiseworthy in an educational context.10.Select the sentence that has the correct use of the present perfect tense.A)She has gone to the store already.B)He is gone to the store since yesterday.C)They have been gone to the movies for two hours.D)We had gone to the park before the rain started.Answer: A) She has gone to the store already.Explanation: The present perfect tense is used to describe an action that started in the past and continues to the present or an action that has a present result. Option A correctly uses the present perfect tense to indicate that the action of going to the store has happened in the past and might still be ongoing. Options B, C, and D are incorrect because they use the past perfect, present perfect continuous, or past perfect continuous tenses, which are not appropriate in this context.11.Choose the correct word to complete the sentence.The teacher asked us to hand in our homework ____________.A. soonB. quicklyC. earlyD. latelyAnswer: B. quicklyExplanation: The word “quickly” is the correct choice because it means to do something with speed or in a short time, which is suitable for the context of handing in homework. “Soon” means at a later time, “early” means before the scheduled time, and “lately” means in recent times, none of which fit the context as well as “quickly”.12.Select the word that does not belong in the following list.1.apple2.banana3.orange4.pear5.grape6.peach7.pearAnswer: 7. pearExplanation: The word “pear” appears twice in the list, which makes it the odd one out. All the other items are unique fruits, while “pear” is repeated.三、完型填空(10分)Title: Oxford Treehouse Edition English Grade 6A Mid-term ExamSection 3: Cloze TestDirections: Read the passage carefully and choose the best answer for each blank.The following story is about a young girl named Lily who loves reading. She often goes to the library to borrow books.Lily was very excited because she had received a special gift from her grandmother. It was a beautiful 1(bookshelf) for her room. She had always wanted a bookshelf to put all her favorite books on.One evening, Lily decided to 2 (organize) her new bookshelf. She started by 3(arrange) her books by genre. She put all her 4(science) books on one shelfand her 5 (adventure) books on another.As she was arranging her books, Lily realized that she had many books to read. She couldn’t wait to dive into her new collection and explore new worlds.1.a. shelf2.a. arrange3.a. arrange4.a. science5.a. adventureAnswer: 1. a. shelf 2. a. arrange 3. a. arrange 4. a. science 5. a. adventure 四、阅读理解(26分)Reading ComprehensionPassage:The British Museum is located in London, England. It is one of the largest and most famous museums in the world. The museum was founded in 1753 and opened to the public in 1759. The British Museum houses a vast collection of art and artifacts from all over the world. It includes items from ancient civilizations like Egypt, Greece, and Rome, as well as from the Middle East and Asia.One of the most famous items in the museum is the Rosetta Stone. The Rosetta Stone is a granodiorite stele inscribed with a decree issued in 196 BC on behalf of King Ptolemy V. The stone is famous because it contains a decree written in three scripts: hieroglyphic, Demotic, and Greek. The discovery of the RosettaStone in 1799 by French soldiers during the Napoleonic Wars helped decipher Egyptian hieroglyphs and opened the way for understanding ancient Egyptian history.Another significant item is the Elgin Marbles, which were taken from the Parthenon in Athens, Greece, by Lord Elgin in the early 19th century. The Parthenon was a temple dedicated to the goddess Athena. The Elgin Marbles include the frieze, metopes, and pediments from the Parthenon. These marble sculptures depict various gods, goddesses, and heroes from Greek mythology.The British Museum also has a large collection of coins, medals, and coins from ancient civilizations. The coins and medals collection provides valuable insights into the economic and political history of various civilizations.Questions:1.When was the British Museum founded?A) 1753B) 1759C) 196 BCD) 17992.What is the significance of the Rosetta Stone?A) It is a granodiorite stele inscribed with a decree.B) It contains a decree written in three scripts.C) It depicts gods, goddesses, and heroes from Greek mythology.D) It was taken from the Parthenon in Athens.3.Which of the following is true about the Elgin Marbles?A) They are a collection of coins and medals.B) They include the frieze, metopes, and pediments from the Parthenon.C) They are from ancient Egyptian history.D) They were taken from the British Museum.Answers:1.A) 17532.B) It contains a decree written in three scripts.3.B) They include the frieze, metopes, and pediments from the Parthenon.五、写作题(16分)Title: My Ideal SchoolWriting Prompt: Imagine a perfect school where everything is just right. Describe your ideal school, including the facilities, teachers, and students. Why do you think such a school would be beneficial for students?Example:My Ideal SchoolIn my ideal school, everything is designed to foster a positive and enriching learning environment. The campus is situated in a serene location, surrounded by lush greenery and a sparkling lake. The buildings are modern and well-equipped, with spacious classrooms, well-stocked libraries, and advanced technology labs.The teachers in my ideal school are highly qualified and passionate about their profession. They are patient, approachable, and dedicated to helping eachstudent reach their full potential. They use innovative teaching methods and encourage students to think critically and creatively. The teachers also organize various extracurricular activities, such as sports, music, and art clubs, to promote holistic development.The students in my ideal school come from diverse backgrounds, but they all share a common goal of learning and growing. They are respectful, cooperative, and supportive of each other. The school culture is inclusive and encourages students to embrace their unique talents and interests.One of the key features of my ideal school is the emphasis on sustainability. The campus is powered by renewable energy sources, and students are encouraged to participate in environmental conservation projects. This not only teaches them about the importance of taking care of our planet but also instills in them a sense of responsibility.I believe such a school would be beneficial for students because it provides them with a well-rounded education that caters to their academic, social, and emotional needs. The supportive environment allows students to thrive and develop into well-rounded individuals who are ready to face the challenges of the future.Analysis:In this example, the student has clearly described their ideal school, covering various aspects such as facilities, teachers, and students. The example demonstrates a strong understanding of the topic and effectively communicatesthe benefits of an ideal school environment. The writer uses vivid and descriptive language to create a picture of the perfect school, and they provide clear reasons for why such a school would be beneficial for students. The example also shows an understanding of the importance of sustainability in education.。

2024-2025学年人教PEP版英语六年级上册期中试题(含答案)

2024-2025学年人教PEP版英语六年级上册期中试题(含答案)

六年级上册英语人教版期中模拟达标卷(1)(满分70分)班级:姓名:成绩:一、单项选择题。

(10分)( ) 1.We will go by _________.A.subway B.a plane C.pizza( ) 2.They’re ________ the zoo.A.bye B.early C.behind( ) 3.I’m going to the __________ tonight. I want to buy some storybooks. A.post office B.bookstore C.museum( ) 4.The drivers must ________ at a red light.A.go B.slow down C.stop and wait( ) 5.—How do you get to the USA from China?—__________.A.By plane B.By subway C.By bus二、选出不同类的单词。

(10分)( ) 1.A.Alaska B.China C.Germany D.Scotland( ) 2.A.get B.by C.buy D.take( ) 3.A.slow B.stop C.wait D.go( ) 4.A.post office B.museum C.postcard D.restaurant ( ) 5.A.sled B.helmet C.ferry D.Bike三、选择与句子相符的图片。

(10分)A.B.C.D. E.( ) 1.—What are you going to do tomorrow?—I’m going to see a film.( ) 2.—Let’s play together.—Sorry. I have to do my homework now.( ) 3.—What are you going to do this afternoon?—I’m going to buy a word book.( ) 4.—What will your aunt do for Mid-Autumn Festival? —My aunt will make mooncakes.( ) 5.—Are you going to the park this Sunday?—Yes. I’m going to draw some pictures.四、连词成句。

小学六年级上册I卷英语第二单元期中试卷(答案和解释)

小学六年级上册I卷英语第二单元期中试卷(答案和解释)

小学六年级上册英语第二单元期中试卷(答案和解释)英语试题一、综合题(本题有50小题,每小题2分,共100分.每小题不选、错误,均不给分)1.Which one is not a fruit?A. OrangeB. PeachC. CucumberD. Pineapple2.What do you use to write?A. BookB. PenC. ChairD. Eraser3.Which of the following is a color?A. AppleB. BlueC. ElephantD. Shoe4.Which of these is a color?A. RedB. TableC. CarD. Phone5.What do we wear on our feet?A. ShoesB. HatC. ScarfD. Gloves6.Which one of these is a fruit?A. CarrotB. BananaC. PotatoD. Onion7.Which of these animals is famous for being slow?A. RabbitB. TurtleC. CheetahD. Elephant8.Which of these is used to write on paper?A. PencilB. PlateC. SpoonD. Fork9.Which of these is a type of tree?A. RoseB. OakC. CarrotD. Strawberry10.What is the color of the sky on a sunny day?A. BlueB. GreenC. BlackD. White11.What is the opposite of "old"?A. NewB. TallC. BigD. Small12.Tom ______ to play football.A. likeB. likesC. likingD. liked13.David is excited to go camping with his family. They pack their __________, __________, and __________ in the car. David also brings a __________ to roast marshmallows over the campfire. They set up their __________ and enjoy a beautiful __________ in the evening.14.Which one is a furniture item?A. SofaB. SpoonC. KnifeD. Plate15.What do we use to cut paper?A. ScissorsB. ForkC. KnifeD. Spoon16.Which of these is a season?A. SummerB. MondayC. TableD. Chair17.Which of these is used to clean the floor?A. BroomB. SpoonC. BookD. Chair18.Which one is correct?A. I have a apple.B. I have an apple.C. I have the apple.D. I have apples.19.Which of the following is a fruit?A. CarrotB. BananaC. BroccoliD. Cucumber20.What do you use to see?A. EarsB. EyesC. MouthD. Nose21.Which animal barks?A. CatB. DogC. CowD. Bird22.What do we use to wash our hands?A. ToothbrushB. SoapC. PenD. Cup23.Which of these is a body part?A. TableB. FootC. ChairD. Book24.Which one is used for cooking?A. ForkB. KnifeC. PanD. Platest weekend, I _______ (have) a sleepover at my friend’s house. We _______ (watch) a movie and then _______ (play) video games. After that, we _______ (make) some snacks and _______ (sit) in the living room talking. My friend _______ (bring) her pet dog, and we _______ (play) with it. We _______ (stay) up late, but we _______ (feel) very happy. In the morning, we _______ (wake) up early and _______ (eat) breakfast together. It _______ (be) so much fun.26.What do you use to cut paper?A. ScissorsB. KnifeC. SpoonD. Plate27.How do you say "你好" in English?A. GoodbyeB. Thank youC. HelloD. Sorry28.Which of these is an insect?A. ElephantB. AntC. FishD. Bird29.I __________ (feel) very tired because I __________ (stay) up late last night. I__________ (try) to get more sleep tonight.30.What do you wear on your feet?A. HatB. GlovesC. ShoesD. Jacket31.Which of these is a fruit?A. CarrotB. AppleC. PotatoD. Onion32.My sister ______ (like) to read books in the evening. She ______ (read) for about 30 minutes before going to bed. Yesterday, she ______ (finish) reading a novel, and she ______ (feel) happy because it ______ (be) a very interesting story. We ______ (talk) about the book after she ______ (finish) it.33.How do you say "你好" in English?A. GoodbyeB. HelloC. Thank youD. Sorry34.Which of these is a number?A. NineB. TableC. SpoonD. Plate35.Which one is a shape?A. SquareB. DogC. ChairD. Book36.Which one is a body part?A. ArmB. SpoonC. TableD. Shoe37.Which of these animals can run fast?A. ElephantB. GiraffeC. CheetahD. Dog38.Which animal can swim?A. DogB. CatC. FishD. Bird39.I __________ (be) a student in Class 5. Every day, I __________ (go) to school with my friends. We __________ (study) many subjects, like math, English, and science. After school, I __________ (like) to play basketball. My best friend, Tim, __________ (join) me sometimes.40.Which of the following is a time of day?A. BreakfastB. MorningC. LunchD. Dinner41.Which one is a wild animal?A. DogB. ElephantC. CowD. Sheep42.Which animal is known for flying?A. DogB. BirdC. FishD. Elephant43.Which one is a shape?A. appleB. squareC. carD. tree44.There ______ a book on the desk.A. isB. areC. beD. am45.Tomorrow, my class ______ (have) a field trip to the museum. We ______ (learn) about dinosaurs and ancient history. I ______ (look) forward to seeing the dinosaur skeletons. I ______ (never) ______ (see) them before.46.Which one is a vegetable?A. BananaB. TomatoC. CucumberD. Mango47.At my school, we have a __________. It is full of __________ and has many__________. I like to go there during __________ because I can choose my favorite__________ to read. The librarian is very __________ and always helps us find the books we need. I enjoy reading __________ books about __________ because they are very interesting.48.Which one is a number?A. FiveB. ChairC. SpoonD. Table49.Which of these is a musical instrument?A. PianoB. SpoonC. PlateD. Chair50.Which of these is a type of food?A. CarB. PizzaC. BookD. Chair(答案及解释)。

期中达标测试卷(二)(试题)-2024-2025学年冀教版版英语六年级上册( 含答案)

期中达标测试卷(二)(试题)-2024-2025学年冀教版版英语六年级上册(  含答案)

期中达标测试卷(二)Unit 1~ Unit 2时间: 60分钟分数: 100分题号一二三四五六七八总分得分一、看图片,写单词。

(10分)1. The girl is in the .2. They are having at school.3. Mr. Li is a .4. I have a new .5. Tom learned many new .二、选出下列每组单词中不同类的一项。

(10分)( )1. A. bathroom B. help C. kitchen D. bedroom ( )2. A. driver B. teacher C. ill D. doctor( )3. A. breakfast B. lunch C. dinner D. table ( )4. A. boy B. Mr. C. Mrs. D. Ms.( )5. A. always B. often C. box D. never三、英汉互译。

(15分)1.在厨房2. a rainy day3.公交车司机4. look out5.校车6. write a letter7.玩纸牌 8. have breakfast9.多少 10. wash the dishes四、单项选择。

(请将正确答案的序号填在题前括号内)(每小题2分,共20分)( )1. There are many in the classroom.A. childB. childsC. children( )2. He is a letter.A. writesB. writingC. writting( )3.— Danny, do you wear dresses? — No,I wear dresses.A. sometimesB. neverC. often( )4. Mary is in the U. S.. I want to send a postcard.B. himC. her( )5. These many photos.A. isB. areC. be( )6. What would you like dinner?A. forB. toC. with( )7. the temperature outside?A. How'sB. whatC. what's( )8. My hands are dirty. I want to them.A. washB. brushC. dry( )9. Mum is cooking in the .A. living roomB. kitchenC. bathroom( )10. Mr. and Mrs. Smith often for a walk after dinner.A. goB. goesC. to go五、用所给词的正确形式填空。

小学六年级上册第十三次英语第三单元期中试卷(答案和解释)

小学六年级上册第十三次英语第三单元期中试卷(答案和解释)

小学六年级上册英语第三单元期中试卷(答案和解释)英语试题一、综合题(本题有50小题,每小题2分,共100分.每小题不选、错误,均不给分)1.Which one is a fruit?A. AppleB. CarrotC. CucumberD. Onion2.Which of these is a time of day?A. MorningB. DinnerC. ForkD. Spoon3.Which one is a pet animal?A. ElephantB. DogC. TigerD. Lion4.Which one is a color?A. RedB. SpoonC. PlateD. Chair5.Which of these is a place to sleep?A. BedB. CarC. ChairD. Table6.Yesterday, I __________ (go) to the zoo with my family. We __________ (see) many animals. My sister __________ (like) the pandas, but I __________ (prefer) the elephants. We __________ (stay) there for three hours, and then we __________ (have) lunch at a café.7.How do you say "再见" in English?A. ByeB. HelloC. Thank youD. Good night8.Mia is at the park. She is riding her __________ (1) and having fun. She sees a__________ (2) and a __________ (3) in the distance. After riding, Mia sits on a__________ (4) to rest.9.Which one is the correct spelling?A. RecieveB. ReceiveC. RecsiveD. Reciive10.This morning, I __________ (wake) up at 6:30. After I __________ (get) up, I__________ (brush) my teeth and __________ (eat) breakfast. Then, I __________ (leave) home to catch the bus.11.What do we use to tell time?A. SpoonB. WatchC. PlateD. Knife12.Which of these is used for eating?A. KnifeB. SpoonC. PlateD. Cup13.Sophia is baking a __________ in the kitchen. She mixes __________, sugar, and butter together in a big bowl. Then, she adds __________ and mixes it again. After that, Sophia pours the dough into a __________ and places it in the oven. When it’s ready, she takes it out and lets it cool before enjoying a delicious __________ with her family.st weekend, we __________ (visit) my grandparents. They __________ (live) ina small village. We __________ (stay) there for two days. My grandmother __________ (make) delicious cookies, and we __________ (eat) them together.15.Which of these is a bird?A. DogB. FishC. CatD. Penguin16.My best friend and I __________ (meet) at school every day. We __________ (sit) together in class, and we __________ (talk) about our favorite books. Last Friday, we__________ (go) to the library to borrow some new books.17.Sarah is very kind. Every morning, she ______ (help) her little brother get ready for school. She ______ (make) him breakfast and ______ (pack) his school bag. Last week, she ______ (buy) him a new notebook because he ______ (need) it for school.18.Which of these is a type of tree?A. OakB. CarrotC. TableD. Dog19.I have a great __. His name is __. He is very __ and loves to play with me. Every morning, we go to the __ together. He runs fast and likes to play __ with other dogs. Afterward, we go back home and eat some __.20.What time is it when the clock shows 3:00?A. Three oclockB. Four oclockC. Five oclockD. Six oclock21.Which one is used for writing?A. PenB. ForkC. KnifeD. Spoon22.What do we use to make a phone call?A. PhoneB. SpoonC. PlateD. Chair23.We ______ (have) a pet rabbit named Snowball. Every morning, I ______ (feed) him some carrots and lettuce. Snowball ______ (like) to hop around the garden, and sometimes he ______ (hide) under the bushes. At night, he ______ (sleep) in his little house. I ______ (love) Snowball very much.24.Which one is a body part?A. HandB. PlateC. ForkD. Spoon25.I have ___ book on my desk.A. aB. anC. theD. some26.My friend Alice __________ (play) the piano very well. She __________ (practice) every day after school. Yesterday, she __________ (perform) in a concert and__________ (sing) a beautiful song. I __________ (go) to the concert with my mom, and we __________ (enjoy) it very much.27.Which of the following is a healthy food?A. CakeB. AppleC. CandyD. Soda28.Which of these is a color?A. DogB. PurpleC. TableD. Spoon29.What color is the sky?A. It is blue.B. It is big.C. It is heavy.D. It is fast.st holiday, my family and I __________ (1) to a new restaurant. The food__________ (2) very delicious. I __________ (3) some pasta and my sister __________ (4) a pizza. After we __________ (5) our meal, we __________ (6) some ice cream for dessert.31.Tom is visiting the __________ with his family. They are looking at the__________ in the trees. Tom’s little brother likes the __________ because they make funny sounds. After visiting the animals, they sit on a __________ and eat some__________.32.How do you greet someone in the morning?A. Good nightB. Good morningC. Good afternoonD. Good evening33.Where is the pencil? It is ______ the box.A. inB. onC. atD. under34.What do you use to write on paper?A. PencilB. SpoonC. PlateD. Shoe35.Which of these is used to cut paper?A. SpoonB. KnifeC. ScissorsD. Fork36.We __________ (not/like) doing chores at home, but it’s something we__________ (have) to do. I __________ (wash) the dishes, and my brother __________ (vacuum) the floor. My parents __________ (tell) us it __________ (be) important to help around the house.37.What do we call a shape that has three sides and three angles?A. CircleB. SquareC. TriangleD. Rectangle38.I love going to the ______ because I can see many different kinds of plants. My mom often buys ______ there for the house. We also like to take photos of the colorful ______ an d beautiful flowers. It’s a peaceful place to spend time.39.I ______ (read) a book about dinosaurs last week. It ______ (be) very interesting. I ______ (learn) a lot of new things about them.40.Which of these is a tool?A. SpoonB. KnifeC. PlateD. Fork41.Which one is a body part?A. LegB. ChairC. PlateD. Table42.Which one is a holiday?A. MondayB. ChristmasC. FridayD. January43.Which animal is known for saying "moo"?A. CowB. SheepC. DogD. Cat44.What is the time when the sun is highest in the sky?A. MorningB. NoonC. AfternoonD. Evening45.Which of these is a season?A. SundayB. MarchC. WinterD. January46.Anna is learning how to __________. She starts by __________ on a small__________. Anna wears a __________ to stay safe. She practices every day until she feels confident and can ride without __________. Her parents are very proud of her.47.Which of these is a pet?A. TigerB. DogC. ElephantD. Lion48.My dad ______ (be) a teacher. Every day, he ______ (teach) students at school. He ______ (like) his job because he ______ (help) kids learn new things. On weekends, he ______ (spend) time with our family.49.What do you use to write on paper?A. SpoonB. PenC. PlateD. Knife50.Which animal lives in water?A. DogB. FishC. CatD. Horse(答案及解释)。

人教PEP版英语六年级上册期中试题(含答案及解析)

人教PEP版英语六年级上册期中试题(含答案及解析)
2023-2024 学年 期中复习训练(试题)人教 PEP 版 英语六年 级上册
一、语音选择题 选出画线部分发音不同的一项
1.
A.song
B.hospital
2.
A.big
B.ill
3.
A.bike
B.pig
4.
A.April
B.cake
C.into C.bike C.dig C.candle
二、词汇选择题 找出不同类的一项。 5.A.tomorrow 6.A.visit 7.A.going 8.A.ship 9.A.by
(safety) on the road.
49.There
(be) no buses on the road.
七、补全句子 50.My father likes to take the s
to work. It's fast and safe.
51.Slow down and s
at a yellow light.
A.me
B.he
21.How do you get to the USA _________ China ?
A.in
B.from
C.to
22.The restaurant is ________ the Palace Museum. ( )
A.next
B.next to
C.next from
23.当你去商店时,售货员通常首先会说: __________ ( )
71.A.In
B.On
C.At
72.A.cinema
B.hospital
C.bookstore
73.A.read
B.watch

六年级英语上册期中考试卷【含答案】

六年级英语上册期中考试卷【含答案】

六年级英语上册期中考试卷【含答案】专业课原理概述部分一、选择题(每题1分,共5分)1. 下列哪个单词的发音与其他不同?A. catB. dogC. fishD. bird2. 选择正确的答案来完成句子:“I ______ to the movies last night.”A. goB. goesC. wentD. gone3. “She ______ her homework every day.” Which word plete s the sentence?A. doB. doesC. didD. done4. Which sentence is correct?A. He like apples.B. He likes apple.C. He likes apples.D. He liking apples.5. What is the past tense of “write”?A. writedB. wroteC. writtenD. write二、判断题(每题1分,共5分)6. “He didn’t go to school yesterday” means he went to school. ( )7. “She can swim” means she is able to swim. ( )8. “I am reading a book” is present continuous tense. ( )9. “They have finished their work” is present perfect t ense. ( )10. “We will travel to Japan next year” is future tense. ( )三、填空题(每题1分,共5分)11. I ________ (to go) to the library tomorrow.12. They ________ (to watch) a movie last night.13. She ________ (to eat) an apple for breakfast.14. We ________ (to travel) to France next summer.15. He ________ (to finish) his homework.四、简答题(每题2分,共10分)16. What is the difference between “I do” and “I am doing”?17. Expln the past perfect tense in a sentence.18. What is the plural form of “child”?19. Write a sentence using “if” clause.20. How do you form a question in English?五、应用题(每题2分,共10分)21. Translate the following sentence into English: “他昨天去了公园。

小学六年级上册第3次英语第四单元期中试卷(答案和解释)

小学六年级上册第3次英语第四单元期中试卷(答案和解释)

小学六年级上册英语第四单元期中试卷(答案和解释)英语试题一、综合题(本题有50小题,每小题2分,共100分.每小题不选、错误,均不给分)1.What is the opposite of "hot"?A. ColdB. WarmC. TallD. Light2.Which of these is a type of food?A. ShoeB. AppleC. ShirtD. Car3.How are you today?A. I am a student.B. I am fine, thank you.C. I am in class.D. I am reading a book.4.I usually ______ (wake) up early in the morning. Today, I ______ (wake) up late because I ______ (stay) up too late last night. I ______ (not have) time for breakfast, so I ______ (eat) a sandwich at school. I ______ (feel) tired, but I ______ (try) my best to pay attention in class.5.What is the opposite of "happy"?A. SadB. TallC. BigD. Slow6.Which of these is a vegetable?A. AppleB. CarrotC. BananaD. Grape7.Molly loves to draw. She always carries her __________ with her, filled with markers, crayons, and colored pencils. Today, she is drawing a picture of a __________. She starts by sketching the body, then adds the legs and tail. After finishing the outline, she begins coloring the __________. Molly carefully chooses bright colors to make her picture look beautiful. When she finishes, she shows it to her mom and proudly hangs it on the __________.8.What is the time? It’s ________ o’clock.A. twoB. secondC. toD. clock9.Which of these is a number?A. DogB. OneC. BookD. Chair10.Which of these is a fruit?A. PotatoB. AppleC. CarrotD. Cucumber11.My dad ______ (work) at a bank. He usually ______ (leave) home at 8:00 and______ (come) back at 6:00. He ______ (like) his job because it ______ (be) interesting.I ______ (want) to be a teacher when I grow up.12.Every day after school, I walk home with my friend Lily. On the way, we pass a big __ where we often see different kinds of flowers. Lily loves to pick __ to give to her mom. We also like to stop by the __ to see the animals. Sometimes, we even see a rabbit hopping around.13.Sally and her friends are visiting the zoo today. They are very excited to see all the animals. First, they go to the __________ to watch the lions sleep in the sun. Then, they walk to the __________ area where they see __________, jumping around. After that, they visit the __________ to see the elephants, and they are amazed at how big the elephants are. Sally’s favorite animal is th e __________, because it has a long neck. st summer, we __________ (go) to the beach for our holiday. We __________ (swim) in the sea, and I __________ (build) a sandcastle. My sister __________ (collect) seashells, and my brother __________ (play) with his kite. We __________ (have) a lot of fun and __________ (enjoy) the sunshine.15.Which of these is a fruit?A. BananaB. CarrotC. LettuceD. Onion16.My sister ______ (be) very good at drawing. She ______ (draw) pictures every day. Last week, she ______ (paint) a beautiful landscape. I ______ (watch) her work, and I______ (feel) amazed by her talent. She ______ (give) me the picture as a gift.17.Which of these is used to clean?A. BroomB. SpoonC. PlateD. Chair18.Which of these is an animal that lives in the water?A. LionB. FishC. BirdD. Tiger19.Which word means "a place where you go to learn"?A. ParkB. SchoolC. HospitalD. Supermarket20.Which one is a color?A. RedB. DogC. CarD. Tree21.Ben loves to read books. Every evening, he sits on his __________ (1) and reads a __________ (2). His favorite book is about __________ (3). Sometimes, Ben draws pictures of the story in his __________ (4). He enjoys reading because it helps him learn about new __________ (5).22.Which of these is used to clean?A. KnifeB. SpoonC. BrushD. Plate23.What do we wear when it is cold?A. TshirtB. SweaterC. ShortsD. Hat24.Sally is baking cookies. She puts some __________ in the bowl and mixes it with __________. After shaping the dough into small balls, she places them on a __________. The cookies are __________ after baking.25.My family and I ______ (go) to the zoo last weekend. We ______ (see) many animals like lions, tigers, and giraffes. The elephants ______ (be) my favorite. We______ (take) a lot of pictures, and then we ______ (have) lunch at the zoo cafe.26.Which of these is used to clean?A. BroomB. KnifeC. PlateD. Fork27.Which one is a classroom object?A. TVB. TableC. SpoonD. Forkst weekend, I went on a trip with my family. We went to a beautiful __. There were tall trees, colorful flowers, and birds singing in the trees. We walked along a path and saw many interesting animals. My brother took lots of pictures with his __. After walking for a while, we had a picnic near the river.29.Ben and his family are going on a road trip. They are going to the mountains for a weekend getaway. Ben packs his __________ (suitcase) with clothes and his __________ (camera). He also takes his favorite __________ (book) to read during the trip. After a long drive, they stop at a rest __________ (stop) to stretch and have a snack.30.Which color is a leaf in summer?A. BlueB. GreenC. YellowD. Red31.What is the color of an orange?A. RedB. GreenC. YellowD. Orange32.My brother __________ (1) to play basketball every afternoon. He __________ (2) his friends to the court, and they __________ (3) a game for an hour. After the game, they __________ (4) some water.33.What is the opposite of "fast"?A. SlowB. QuickC. FastestD. Speed34.You are at the beach. The sun is shining, and you are swimming in the ocean. What is the weather like?A. SnowyB. WindyC. Hot and sunnyD. Cold and rainyst weekend, my family and I ______ (visit) my grandmother’s house. We ______ (have) a big family dinner together. My grandmother ______ (cook) chicken, and my mom ______ (make) a salad. After dinner, we ______ (play) board games and ______ (watch) a movie. It ______ (be) a wonderful day.36.Next Monday, we ______ (have) a field trip to the zoo. Our teacher ______ (say) that we ______ (see) many animals, including lions, tigers, and elephants. I ______ (bring) my camera to take pictures. We ______ (meet) at school at 9 AM before we______ (leave).37.I love going to the zoo with my family. Yesterday, we __________ (1) to see the new panda. The panda __________ (2) in a big cage, and it __________ (3) bamboo all day. We __________ (4) pictures of it, and my little brother __________ (5) to watch the panda climb trees. After visiting the panda, we __________ (6) to the lion’s area. The lions __________ (7) on the rocks, looking very relaxed. It __________ (8) a fun day at the zoo!38.My father _______ (work) in a bank, and my mother _______ (teach) at a school. They both _______ (work) very hard, but they always _______ (make) time for me and my brother. We love spending time together on weekends.39.What is the opposite of “up”?A. LeftB. RightC. DownD. Forward40.Which animal is known for saying "meow"?A. DogB. CatC. CowD. Sheep41.It’s a rainy day, so we decided to stay inside. My sister and I played a game of __ while our mom made __ for lunch. We also watched a movie about __ and ate some __. It was a cozy and fun day.42.Tom is visiting his grandmother. She has a __________ (1) in her backyard. Tom helps her water the __________ (2) and pick the __________ (3). After working in the garden, they have some __________ (4) together.43.Jack is preparing for his birthday party. He has already picked out a __________, and he is excited to __________ it to his friends. He also helps his mom set the table with __________, napkins, and cups. When his friends arrive, they play __________ and eat some __________. Jack feels happy and thanks everyone for coming to his special day.44.What do we use to clean our teeth?A. ToothbrushB. SpoonC. PlateD. Hatst summer, my family and I __________ (1) to the beach for vacation. We__________ (2) in a small hotel near the sea. Every day, we __________ (3) swimming and __________ (4) ice cream. I __________ (5) to visit the beach again next summer.46.Which animal can fly?A. ElephantB. BirdC. FishD. Dog47.Which of the following is an animal?A. AppleB. CarC. DogD. Table48.What do we use to clean our teeth?A. ForkB. SpoonC. KnifeD. Toothbrush49.Which of these is used to comb hair?A. BrushB. KnifeC. PlateD. Spoon50.Which animal is known for its ability to swim?A. DogB. LionC. FishD. Tiger(答案及解释)。

六年级英语上册期中试卷【含答案】

六年级英语上册期中试卷【含答案】

六年级英语上册期中试卷【含答案】专业课原理概述部分一、选择题(每题1分,共5分)1. 下列哪个单词的发音与其他不同?A. catB. dogC. fishD. bird2. 选择正确的答案完成句子:“I _______ to the park yesterday.”A. goB. goesC. wentD. gone3. “He is _______ the classroom.” 的正确选项是:A. inB. onC. atD. to4. 下列哪个单词意为“快乐的”?A. sadB. happyC. angryD. tired5. “She _______ her homework every day.” 的正确选项是:A. doB. doesC. doingD. did二、判断题(每题1分,共5分)1. English is the most spoken language in the world. ( )2. “I like apples” is a correct sentence. ( )3. “He am a teacher” is a correct sentence. ( )4. “She goes to school bus” means she walks to school. ( )5. “Cat” and “Dog” are both nouns. ( )三、填空题(每题1分,共5分)1. I _______ (be) a student.2. She _______ (have) a book.3. We _______ (go) to the movies last night.4. He _______ (like) playing soccer.5. They _______ (be) happy.四、简答题(每题2分,共10分)1. What is the past tense of “go”?2. Write a sentence using “because”.3. What is the opposite of “big”?4. How do you spell “elephant”?5. What is the plural of “child”?五、应用题(每题2分,共10分)1. Translate: “Elle me manger des pommes.”2. Fill in the blanks: “I _______ (to eat) a sandwich.”3. Correct the sentence: “He don’t like apples.”4. Make a question: “She is reading a book.”5. Write a sentence using “always”.六、分析题(每题5分,共10分)1. Expln the difference between “like” and “love”.2. How do you form the present continuous tense?七、实践操作题(每题5分,共10分)1. Write a short dialogue between two friends meeting for the first time.2. Describe your favorite animal using at least 5 adjectives.八、专业设计题(每题2分,共10分)1. 设计一个简单的英语学习计划,包括每天的学习内容和目标。

鲁教版(五四制)六年级英语上册期中综合测试卷含答案

鲁教版(五四制)六年级英语上册期中综合测试卷含答案

鲁教版(五四制)六年级英语上册期中综合测试卷限时:120分钟满分:100分听力部分(20分)一、听句子, 选择正确的应答语, 每个句子读一遍(每小题1分, 共5分)1. A. Beef noodles. B. Yes, I like them. C. No, I don't.2. A. He is popular. B. He's short. C. He likes reading.3. A. I played basketball. B. I went fishing. C. It was excellent.4. A. Yes, I did. B. Yes, they are. C. It was great.5. A. She went to a farm. B. Gina did.C. I did my homework.二、听五段对话, 选择正确答案, 每段对话读两遍(每小题1分, 共5分)6. Does Lily agree to watch a football game?A. Yes, she does.B. No, she doesn't.C. She's very busy.7. What is the woman's sister?A. She is a nurse.B. He is a doctor.C. She is a doctor.8. How does Han Mei usually come to school?A. By bus.B. On foot.C. By car.9. What are they going to do this Sunday?A. Go hiking.B. Go boating.C. Go swimming.10. What's near here?A. A post office.B. A hotel.C. A hospital.三、听两段长对话, 选择正确答案, 每段对话读两遍(每小题1分, 共5分)听第一段对话, 回答第11、12小题。

(PEP)小学六年级英语上册期中测试卷(含答案)

(PEP)小学六年级英语上册期中测试卷(含答案)

小学六年级英语上册期中测试卷【含答案】(满分:100分时间:60分钟)一、按要求写词语。

(10分)1、have (第三人称单数形式) _______ 2.take (过去式) _______ 3.come (现在分词) _______ 4.can’t (完全形式) _______ 5.know (同音词) _______二、选出下列每组单词中不同类的一项,画__线。

(10分)1、A.father B.mother C.apple 2.A.basketball B.worker C.football 3.A.library B.chocolate C.mooncake 4.A.tiger B.wood C.lion 5.A.Earth B.China C.Canada三、选择填空。

(20分)1、—______ are you going?()—Next week.A.What B.When C.How2、_________an interesting story! ()A.What B.How C.How a D.When3、My mother never _______ basketball because she can't play it.()A.plays the B.play C.plays4、We can’t see ____ sun at night. ()A.a B.an C.the D.no5、What Linda do yesterday?()A.is B.does C.did6、—______ the teachers' office? ()—It's on the second floor.A.Where B.Where's C.What's7、There are five _______ on the table. ()A.tomatos B.pinaoes C.radioes D.photos 8、—________ older, Mary or Ann? ()—Mary.A.Whose is B.Who is C.Who are9、February is the __________ month of a year.()A.second B.two C.three10、They are my _______. ()A.friend B.a friend C.friends四、句型转换,按要求完成下列句子。

  1. 1、下载文档前请自行甄别文档内容的完整性,平台不提供额外的编辑、内容补充、找答案等附加服务。
  2. 2、"仅部分预览"的文档,不可在线预览部分如存在完整性等问题,可反馈申请退款(可完整预览的文档不适用该条件!)。
  3. 3、如文档侵犯您的权益,请联系客服反馈,我们会尽快为您处理(人工客服工作时间:9:00-18:30)。

六年级英语上册期中试题(最新版)
六年级英语上册期中试题(最新版)
一.根据听到的内容写单词(6分)
二.听音,根据听到的内容选择相应的选项(5分)
()1.A.tonightB.tomorrowC.todayD.together
()2.A.bookstoreB.libraryC.museumD.hospital
()3.A.subwayB.busC.bikeD.ship
()4.A.tripB.trafficC.trainD.travel
()5.A.farB.fastC.carD.ask
三、听对话,判断下列句子的正误,,对的在括号内写T,错的在括号内F.(10分)
()1.JohnisgoingtobuyatoybusonSaturday.
()2.Johnisgoingtobuyatoycarforhisfriend.
()3.JohnisgoingtothepostofficeonSunday.
()4.JohnisgoingtopostaletteronSundaymorning.
()5.Johnisgoingtothepostofficebysubway.
四、听音选出合适的答语。

(10分)
()1.A.I’mgoingtobuyabook.B.Igotoschoolonfoot.
()2.A.Bybus.B.No,Idon’t.
()3.A.It’snearthehospital.B.Thisafternoon.
()4.A.Yes,heretheyare.B.Yes,itis.
()5.A.It’seastofthecinema.B.Onfoot.
五、听音填空。

(9分)
A:Whatareyougoingtodo_____school?
B:Iwantto______abook.______isthe?
A:It’s__________________theshoestore.
B:HowcanIgettotheshoestore?
A:YoucangobytheNo.15bus.Getoffatthecinema.Then_______and walk_______forthreeminutes.Theshoestoreisonthe______.
笔试部分(60分)
一.读一读,找出每组单词中不同类的一个。

(5分)
()1.A.postofficeB.cinemaC.bookstoreD.left
()2.A.shipB.taxiC.trainD.foot
()3.A.eveningB.tonightC.nextweekD.seeafilm
()4.A.turnB.slowC.officeD.stop
()5.A.whatB.whenC.whereD.with
二、按要求填空(6分)
1、infrontof(反义词)_________
2、near(反义词)__________
3、cross(现在分词)_________
4、one(序数词)_________
5.star(复数形式)________
6.go(三单形式)_________
7.到达(英语)_________8.where(同音词)_________
9.去旅行(英语)_________10.慢下来(英语)_________
11.I(宾格)_________12.payattentionto(汉语)_________
三、选词填空(10分)
()1.Slowdownandstop____aredlight.
A.in
B.on
C.at
()2.I_____gotoschoolbybike.ButsometimesIgobybus.
A.sometimes
ually
C.today
()3.Igotothepark____foot.
A.by
B.on
C.in
()4.--Let’stothecinema.–Great.
A.go
B.to
C.goes
()5.--doyougotoschooleveryday?--Bybike.
A.What
B.How
C.When
()6.--Whatareyougoingtodotomorrow?
--I’mgoingtohaveartlesson.
A.a
B.anCthe
()7.I’mgoingtothesupermarketmysister.
A.of
B.with
C.and
()8.--areyougoingtothebookstore?
--Thisafternoon.
A.How
B.What
C.When
()9.—Whatareyougoingtotomorrow?—Acomicbook..
A.buy
B.do
C.be
()10.--HowAmygototheUK?
A.is
B.do
C.does
四、按要求完成句子(9分)
1.Isthereacinemanearhere?(作肯定回答)
2.what,to,do,you,going,are,school,after,?(连词成句)
3.Thehospitalisnearthepostoffice.(就划线部分提问)
4.Whereareyougoingtodothisweekend?(改错)
5.Youcangobybike.(一般疑问句)
6.Howdoyoucometoschool?(看图回答)
五.从方框中选择合适的句子补全对话,有两项多余。

(5分)
A:Excuseme.B:Yes,thereis.
A:B:It’snexttothecinema.
A:Isthelibraryfar?B:.
A:
B:Youcangoonfoot.Gostraight,thenturnleft.Youcanseeit
A:B:You’rewelcome.
六、阅读理解(20分)
(一)
Hello!MynameisMike.IamanAmericanboy.I’m12yearsold.Istud yinYouyiSchoolinBeijing.Myhomeisnear.Iusuallygotoschoolonfo ot.SometimesIgobybus.Myfatherandmymotherareteachers.Theywor kinamiddleschool.It’sveryfar.Theygotoworkbysubway.Theywork veryhard.Ilovethem.MygrandparentsareintheUSA.Wearegoingtovi sitthembyplanenextyear.
阅读短文,选择正确答案将其代号填入题前括号内。

(10分)
()1.HowoldisMike?
A.Heiseleven.
B.Heistwelve.
C.Heisten.
()2.IsMikeanEnglishboy?
A.Yes,heis.
B.Idon’tknow.
C.No,heisn’t
()3.Howdoesheusuallygotoschool?
A.Heusuallygoestoschoolonfoot..
B.Heusuallygoestoschoolbybus.
C.Heusuallygoestoschoolbycar.
()4.Dohisparentsgotoworkbybus?
A.Yes,theydo.
B.No,theydon’t.
C.Yes,hedoes.
()5.HowaretheygoingtoAmerica?
A.TheyaregoingtoAmericabyplane.
B.TheyaregoingtoAmericabysubway.
C.heyaregoingtoAmericabytrain.
(二)
TodayisMonday.JackgoestoLondontoseeMrBrown.MrBrown’so ff iceisnearthestation,nexttoahospital.Butwhereisthehospital?J ackcan’tfindit.Awomansays,“Gostraightfortenminutes.ThenTu rnleftandthehospitalisontheright.”Jackfindstheplace.
阅读短文,判断正误,对的写“T”,错的写“F”。

(10分)
()1、TodayisSunday。

()2、JackgoestoLondontoseeMrBrown.
()3、MrBrown’sofficeisnearthehospital.
()4、Thewomanknowsthehospital.
()5、Thehospitalisontheleft.
七.作文(5分)
以“Mybusyweekend”为题写一篇小文章。

要求:字迹工整,语句通顺,不少于5句话。

Mybusyweekend
I’mgoingtohaveabusyweekend.。

相关文档
最新文档